Jump to content

MoMo123's Content

There have been 190 items by MoMo123 (Search limited from 04-06-2020)



Sort by                Order  

#704675 Cho x,y,z $\geq$0, x+y+z=3. CMR: $2(x^{2}+y^{2}+z^{2})+xy...

Posted by MoMo123 on 01-04-2018 - 09:57 in Đại số

Thực ra bài này có 1 cách khác : Áp dụng BĐT quen thuộc $ x^2+y^2+z^2+2xyz+1\geq 2(xy+yz+zx)$

$\Rightarrow 2(x^2+y^2+z^2)+2xyz-1 \geq (x+y+z)^2-2=7$

Mặt khác $xyz \leq 1 -> xyz\geq 2xyz-1$

$\Rightarrow 2(x^2+y^2+z^2)+xyz \geq 2(x^2+y^2+z^2)+2xyz-1 \geq 7$

$ Q.E.D$




#704637 Đề thi chọn HSG lớp 12 tỉnh Quảng Bình năm 2017-2018

Posted by MoMo123 on 31-03-2018 - 21:38 in Thi HSG cấp Tỉnh, Thành phố. Olympic 30-4. Đề thi và kiểm tra đội tuyển các cấp.

Chỗ này có vấn đề ???!!! $\Rightarrow a^{2}+b^{2}+c^{2}\geq a^{2}b+b^{2}c+c^{2}a$

Thực ra nó xuất phát từ BĐT $\sum a^2b \leq \frac{(a+b+c)(a^2+b^2+c^2)}{3}$




#704357 Cho 3 số x,y,z > 1 thỏa mãn:x+y+z=xyz

Posted by MoMo123 on 26-03-2018 - 20:00 in Bất đẳng thức và cực trị

Câu bđt trong này https://diendantoanh...an/#entry698466




#703854 $\frac{1}{(a-b)^2}+\frac{1}...

Posted by MoMo123 on 18-03-2018 - 17:56 in Bất đẳng thức và cực trị

Mình có cách này không biết có được không

Không mất tính tổng quát, giả sử$ a=Min{{a,b,c}}$

Đặt $b=x+a$

       $c=a+y$

$\rightarrow b-c=x-y$

$ab+bc+ca=3a^2+2a(x+y)+xy\geq xy$

$\rightarrow VP\leq \frac{4}{xy}$

-> Ta cần chứng minh

$\frac{1}{x^2}+\frac{1}{y^2}+\frac{1}{(x-y)^2} \geq \frac{4}{xy}$

$\Leftrightarrow \frac{x}{y}+\frac{y}{x}+\frac{xy}{(x-y)^2} \geq 4$

$\Leftrightarrow \frac{x}{y}+\frac{y}{x}+\frac{1}{\frac{x}{y}+\frac{y}{x}-2}\geq 4$

Đến đây chắc bạn tự làm được rồi :)




#703697 Đề thi HSG 9 tỉnh Nghệ An năm học 2017-2018

Posted by MoMo123 on 16-03-2018 - 19:25 in Tài liệu - Đề thi

Đề lần này nói chung không khó lắm vừa sức
Câu tổ :
Từ 1điểm nằm trong đa giác, nối các đoạn thẳng đến các đỉnh của đa giác ta được 2018 tam giấc không có điểm trong chung
Xét điểm thứ 2
Nếu điểm ấy nằm trong 1 tam giác nào đó trong các tam giác đã nêu trên, ta nối các đoạn thẳng đến các đỉnh của tam giác đó , số tam giác không có đỉnh trong chung tăng thêm 2
Nếu điểm ấy nằm trên 1 trong 2018 đoạn thẳng nói trên , ta nối các đoạn thẳng đến các đỉnh đối diện với đoạn ấy thì số tam giác không có điểm trong chung cũng tăng 2
Nên từ đây ta suy ra số tam giác không có điểm trong chung sau 2017 lần làm như vậy là 2018+2x2016=6050
Từ đây ta suy ra tồn tại một tam giác có đỉnh là một trong các điểm trên có diện tích không quá $\frac{1}{6050}$

Câu bất thì biến đổi thành 3P $\leq \sum \frac{xy}{x+y+xy}$ sau đó chia xuống
rồi đặt $\frac{1}{x} =a^3 ;\frac{1}{y}=b^3 ;\frac{1}{z}=c^3 \rightarrow abc =1 $
Đưa về bài toán cơ bản




#703193 $p^{q-1}+q^{p-1}$

Posted by MoMo123 on 10-03-2018 - 12:30 in Số học

Mình làm thế này không biết có đúng không:
+) Xét $\left\{\begin{matrix}p=2m+1 \\ q=2n+1 \end{matrix}\right. (m,n\epsilon N*)$
Đặt $p^{q-1}+q^{p-1}=a^{2}(a\epsilon N)$
$=>(2m+1)^{2n}+(2n+1)^{2m}=a^{2}(**)$
$=>(2m+1)^{2n}=\left [ a-(2n+1)^{m} \right ]\left [ a+(2n+1)^{m} \right ]$
Goi $d=(a-(2n+1)^{m},a+(2n+1)^{m}) =>\left\{\begin{matrix}a-(2n+1)^{m}\vdots d \\ a+(2n+1)^{m}\vdots d \end{matrix}\right. =>\left\{\begin{matrix}2a\vdots d \\ a+(2n+1)^{m}\vdots d \end{matrix}\right.$
Do $(2m+1)^{2n}$ lẻ nen $a-(2n+1)^{m},a+(2n+1)^{m}$ le.
=> $(2,d)=1=>a\vdots d=>(2n+1)^{m}\vdots d=>\left [ a-(2n+1)^{m} \right ]\left [ a+(2n+1)^{m} \right ]\vdots d^{2}=>(2m+1)^{2n}\vdots d^{2}=>(2m+1)^{n}\vdots d$
$=>\left\{\begin{matrix}(2m+1)^{n}\vdots d \\ (2n+1)^{m}\vdots d \end{matrix}\right. => \left\{\begin{matrix}q^{m}\vdots d \\ p^{n}\vdots d \end{matrix}\right. =>d=1 (1)$
=> $a-(2n+1)^{m},a+(2n+1)^{m}$ la SCP $(2)$
Do $2m+1$ la SNT nen $U((2m+1)^{2n})\epsilon \left. 1,2m+1,(2m+1)^{2},...(2m+1)^{2n} \right \}$
Kết hợp với (1) và (2) $=>\left\{\begin{matrix}a-(2n+1)^{m}=1(*) \\ a+(2n+1)^{m}=(2m+1)^{2n} \end{matrix}\right.$
$(*)=>a-q^{m}=1$
Tương tự $a-p^{n}=1=>q^{m}=p^{n}=>q^{2m}=p^{2n}$ thay vao (**)
$=>2q^{2m}$ la SCP (vo ly)
+) Xét chỉ tồn tại một số chẵn trong p,q. Không mất tính tổng quát giả sử $q=2$
=> $p+2^{p-1}=a^{2}=>p+2^{2m}=a^{2}=>p=(a-2^{m})(a+2^{m})=>\left\{\begin{matrix}a-2^{m}=1 \\ a+2^{m}=p \end{matrix}\right. =>2.2^{m}+1=p=>2.2^{m}+1=2m+1(=p)=> m=2^{m}$ (cái này vô nghiệm VT<VP)
CM: +) Với $m=1$ $=>1<2$ (t/m)
+) Giả sử mệnh đề đúng với $m=k\epsilon N^{*}$ $=>k< 2^{k}$

+) Xét $p=q=2$ (t/m)


Mình nghĩ Cái chỗ trường hợp 1 không cần phức tạp thế
$(2m+1)^{2n}+ (2n+1)^{2m} $ chia 4 dư 2 thì không thể là số chính phương thì sẽ suy ra không thể là số chính phương



#702716 Đề thi học sinh giỏi môn toán 9 tỉnh Hưng Yên năm học 2017 - 2018

Posted by MoMo123 on 04-03-2018 - 00:15 in Tài liệu - Đề thi

Cách giải khác của bài tổ

Ta loại đi những số chính phương có trong dãy, ta sẽ chứng minh trong các số còn lại nếu chọn ra 311 số thì luôn tồn tại 2 số có tổng =625

Xét 311 số $a_{1};...a_{311}$ Không mất tính tổng quát giả sử $a_{1} >a_{2}>....>a_{311}>0$

Xét 1 dãy số tiếp theo là $0<625-a_{1}<625-a_{2}<...<625-a_{311}$

Nhận thấy 2 dãy trên có tổng số số hạng là 622

Mà các số hạng trên không nhận các giá trị $225;400;625;49;576$

Thật vậy , các số dãy 1 thì chắc chắn không thể rồi, nếu 1số nào đó ở dãy 2 nhận các giá trị trên 

Giả sử đó là$625-a_{i}$ chắc chắn không thể =625

Nếu $625-a_{i}=400 \Rightarrow a_{i}=225 =15^2 $ (mâu thuẫn )

Nếu $625-a_{i}=225 -> a_{i}= 20^2$ (mâu thuẫn)

Vậy 622 số trên chỉ nhận 621 giá trị -> Tồn tại 2 số $a_{i}; a_{j} $ có tổng =625 (mâu thuẫn)

-> ...




#700865 $\large \overline{abc}=a!+b!+c!$

Posted by MoMo123 on 28-01-2018 - 11:16 in Số học

Tìm các chữ số a,b,c thỏa mãn:
$\large \overline{abc}=a!+b!+c!$
(145=1!+4!+5!)

Có thể làm như sau:
$a;b;c \leq 6 \Rightarrow abc \leq 666$ ->
$a!+b!+c! \leq 666 \Rightarrow a;b;c \leq 5$ $->abc \leq 360 $
$->a\leq 3$ $\Rightarrow a!+b!+c!\leq 3!+5!+5! =246 \Rightarrow abc \leq 246$
-> $a\leq 2$ Đến đây xét 2 trường hợp là cả 2 số =5 hoặc có ít nhất 1 số <5 là đươch



#700414 ĐỀ THI HỌC SINH GIỎI LỚP 9 VÒNG 3 NĂM HỌC 2017-2018 HUYỆN THANH CHƯƠNG

Posted by MoMo123 on 17-01-2018 - 17:58 in Tài liệu - Đề thi

unnamed (3).png

P/s: Câu 2a đề cần sửa lại là GTLN nhé các bạn




#699859 Tìm Min của $P=(1+\frac{2}{1-a})(1+\frac...

Posted by MoMo123 on 06-01-2018 - 21:00 in Bất đẳng thức và cực trị

Cho $a,b,c >0$ và $a+b+c=1$. Tìm Min của $P=(1+\frac{2}{1-a})(1+\frac{2}{1-b})(1+\frac{2}{1-c})$

Ta có bđt $ (1+a^{3})(1+b^{3})(1+c^{3}) \geq (1+abc)^{3}$

Áp dụng vào bài toán 

-> $(1+\frac{2}{b+c})(1+\frac{2}{a+c})(1+\frac{2}{a+b}) \geq $ ( $\left ( 1+2\sqrt[3]{\frac{1}{(a+b)(b+c)(c+a)}} \right )^{3}$$ \geq (1+3)^{3}$




#699049 Đề cử Thành viên nổi bật 2017

Posted by MoMo123 on 27-12-2017 - 22:09 in Thông báo tổng quan

Tên nick, ứng viên: chanhquocnghiem, manhtuan00, ecchi123

Thành tích (đóng góp) nổi bât: $+$ chanhquocnghiem: Giúp các thành viên rất tích cực, đặc biệt là các anh chị đang ôn thi đại học

                                                 $+$ ecchi123, manhtuan00: Là 2 thành viên tích cực tham gia giải bài trong chuyên mục " Mỗi tuần một bài Toán hình học''




#698992 ĐỀ THI HỌC SINH GIỎI LỚP 9 VÒNG 2 NĂM 2017-2018 HUYỆN THANH CHƯƠNG NGHỆ AN

Posted by MoMo123 on 27-12-2017 - 12:48 in Tài liệu - Đề thi

Câu 1:

a) Từ giả thiết ta có thể đặt :$n^2-1=3m(m+1)$ với $m$ là một số nguyên dương 

Biến đổi phương trình thì ta có:$(2n-1)(2n+1)=3(2m+1)^2$

Do $(2n-1;2n+1)=1$ nên dẫn đến $2n-1=3u^2;2n+1=v^2$ hoặc $2n-1=u^2;2n+1=3v^2$

Với trường hợp đầu suy ra $v^2-3u^2=2 \Rightarrow v^2 \equiv 2(mod 3)$ (Vô lý)

Còn lại trường hợp thứ hai cho ta $2n-1$ là số chính phương

b) Biến đổi phương trình thì ta có:

$(2x-5)(2y-1)=2k+3$

Nhận thấy rằng $2x-5,2y-1>0$ nên số nghiệm bài toán trên chính là số ước nguyên dương của $2k+3$

Giả sử $2k+3$ có dạng $p_1^{m_1}.p_2^{m_2}...p_n^{m_n}$ thì số ước của nó là $(m_1+1)(m_2+2)...(m_n+1)$

Theo đề bài suy ra $(m_1+1)(m_2+2)...(m_n+1)$ lẻ nên $m_1;m_2;...;m_n$ là các số chẵn

Khi đó $2k+3$ là số chính phương.Dễ kiểm tra số nguyên dương $k$ nhỏ nhất thỏa điề đó là $k=3$

Ta thu phương trình $(2x-5)(2y-1)=9$ và giải ra tìm được $(x;y)=(3;5);(4;2);(7;1)$

Thử là được rồi, dùng hàm $\tau$ làm gì cho nó phức tạp ;)

Làm bài hình nào

geogebra-export (1).png

Câu a, b chắc mọi người làm hết rồi đúng không

CâU C

Theo câu b, ta có : $\frac{KH}{ID}=\frac{KN}{IM} =\frac{KH}{IK} $

 

$IK^{2}=IH.AI -> \frac{IH}{IK}=\frac{IK}{AI}$

-> $\frac{AK}{AI}=\frac{HK}{IK}$ -> $\frac{KN}{IM}=\frac{AK}{AI}$

-> $A,N,M$ thẳng hàng




#698721 $\boxed{\text{TOPIC}}$ Ôn thi học si...

Posted by MoMo123 on 21-12-2017 - 21:23 in Tài liệu - Đề thi

Chà, lâu nay $TOPIC$ trầm ghê ta. Xin lỗi mọi người vì lâu nay mình bận ôn thi nên không giữ lửa cho $TOPIC$ được, mình xin đưa ra một số bài mới sau đây:

$\boxed{\text{Bài 1}}$  Cho $m,n$ là 2 số tự nhiên sao cho $\sqrt{7}-\frac{m}{n} >0$

Chứng minh rằng $n\sqrt{7}-m>\frac{1}{m}$

$\boxed{\text{Bài 2}}$ Tìm tất cả các số nguyên tố $p_{i} ( i \in {1,2,..8}) $ biết

$ \sum_{i=1}^{7}p_{i}^{2}=p_{8}^{2}$

$\boxed{\text{Bài 3}}$ Giả sử $a_{i}, i=\overline{1,17} $ là $17$ số tư nhiên đôi một khác nhau và $5\leq a_{i} \leq 2018 \forall i= \overline {1,17}$. Chứng minh rằng $\exists 9$ số trong chúng sao cho tổng của $9$ số này $\vdots 9$

$\boxed{\text {Bài 4}}$ Cho$a,b,c>0$ thỏa mãn $\frac{1}{a}+\frac{2}{b}+\frac{3}{c}=3$ Chứng minh 

$\frac{27a^{2}}{c(c+9a^{2})}+\frac{b^{2}}{a(4a^{2}+b^{2}}+\frac{8c^{2}}{b(9b^{2}+4c^{2})} \geq \frac{3}{2}$




#698466 ĐỀ THI HỌC SINH GIỎI LỚP 9 VÒNG 2 NĂM 2017-2018 HUYỆN THANH CHƯƠNG NGHỆ AN

Posted by MoMo123 on 17-12-2017 - 17:44 in Tài liệu - Đề thi

1513507558903-1958573039.jpg



#697995 Đề thi hsg huyện Toán lớp 9 Huyện Hưng Nguyên

Posted by MoMo123 on 09-12-2017 - 13:21 in Tài liệu - Đề thi

Mọi người xem đề ở đây cho khỏi  ''gãy cổ'' https://diendantoanh...-1512797276.jpg

Bài bất thì giống đề này https://diendantoanh...attach_id=32960

Bài tổ :

Chia hình vuông thành 16 hình vuông nhỏ bằng nhau, có $R=\frac{1}{4\sqrt{2}} \Rightarrow \exists $ $\left [ \frac{33}{16}\right ]+1 =3 $ điểm cùng $\in$ 1 hình vuông 

$\Rightarrow \exists (O;\frac{1}{5})$  chứa ít nhất 3 điểm trong 33 điểm đã cho




#697558 Cho a,b,c >0 và a+b+c=1. Tìm Min S = 2(a2+b2+c2) + $\frac...

Posted by MoMo123 on 01-12-2017 - 16:29 in Bất đẳng thức và cực trị

$(a+b+c)^{2} \geq 3(ab+bc+ca) <=> (a+b+c-3)(a+b+c-ab-bc-ca)\geq 0 $=>$ a+b+c \geq ab + bc + ca$

Mình không nghĩ đoạn này đúng, Bạn xem lại cách khai triển của bạn xem, Nếu như tách nó ra khỏi bài toán, cho $a+b+c=4$ , $ab+bc+ca=5$ thì mặc dù BĐT trước đúng nhưng $ a+b+c  < ab+bc+ca $ , Đây chính là bài toán BĐT trong VMO-1996 và cũng đã được đề cập tại đây (Bài 2)




#697226 Cho đa giác đều (H) có 14 đỉnh.CMR trong 6 đỉnh bất kì của (H) luôn có 4 đỉnh...

Posted by MoMo123 on 26-11-2017 - 17:46 in Toán rời rạc

Cho đa giác đều (H) có 14 đỉnh.CMR trong 6 đỉnh bất kì của (H) luôn có 4 đỉnh của 1 hình thang

Đã có lời giải tại đây




#697146 TRONG CÁC TẤM BÌA DƯỚI ĐÂY, MỖI TẤM CÓ MỘT MẶT GHI MỘT CHỮ CÁI

Posted by MoMo123 on 24-11-2017 - 22:03 in Toán rời rạc

    Trong các tấm bìa trình bày dưới đây, mỗi tấm có một mặt ghi một chữ cái và mặt kia ghi một số:

                              A       M      3      6

    Chứng tỏ rằng đề kiểm tra câu sau đây có đúng không: " Nếu mỗi tấm bìa mà mặt chữ cái là nguyên âm thì mặt kia là số chẵn", thì chỉ cần lật mặt sau của tối đa là 2 tấm bìa, đó là 2 tấm bìa nào ?

Đề thi học sinh giỏi Toán 9 Thừa Thiên Huế 2006-2007




#696308 ĐỀ THI HỌC SINH GIỎI LỚP 9 VÒNG 1 NĂM 2017-2018 HUYỆN THANH CHƯƠNG NGHỆ AN

Posted by MoMo123 on 10-11-2017 - 19:47 in Tài liệu - Đề thi

 post-165577-0-69765100-1510318775.jpg

 

Đề chuẩn mọi người xem ở đây




#695735 $a,b,c> 0,c> a$

Posted by MoMo123 on 28-10-2017 - 22:33 in Bất đẳng thức và cực trị

$a,b,c> 0,c> a$ chứng minh rằng:
$$\left (\frac{a}{a+b}  \right )^{2}+\left (\frac{b}{b+c}  \right )^{2}+4\left (\frac{c}{c+a}  \right )^{2}\geq \frac{3}{2}$$

Đây chính là câu bất trong đề thi  tuyển sinh PBC năm nay, đã có lời giải tại đây  :)




#695151 $\sum\dfrac{1}{(2x+y)(2x+z)}\ge 1$

Posted by MoMo123 on 21-10-2017 - 11:10 in Bất đẳng thức và cực trị

Cho $x,y,z>0$ thỏa mãn $xy+yz+zx=1$. Tìm giá trị nhỏ nhất của biểu thức sau $$P=\dfrac{1}{(2x+y)(2x+z)}+\dfrac{1}{(2y+x)(2y+z)}+\dfrac{1}{(2z+y)(2z+x)}$$

Em có cách này không biết có đúng không
$\sum \frac{1}{(2x+y)(2x+z)}=\sum \frac{yz}{(2xz+yz)(2xy+yz)}\geq \sum \frac{4yz}{(2xy+2yz+2zx)^{2}}=\sum yz =1$

Bạn giỏi quá, đề nguyên mẫu là như này $P=\sum\dfrac{1}{4x^2-yz+2}$. Bạn có ý tưởng nào khác không?

Lậu lâu làm lại nó cho vui chứ nhỉ anh, một ý tưởng khác của em là dồn biến nhưng cũng không khác mấy cách ban đầu

$x=\frac{1-yz}{y+z}$

$\rightarrow \sum\frac{1}{4x^{2}-yz+2} =\sum \frac{1}{4.(\frac{1-yz}{y+z})^{2}-yz+2}\geq\sum\frac{1}{\frac{(1-yz)^{2}}{yz}-yz+2}=\sum yz=1$




#695137 Cho các số thực dương x, y, z thỏa mãn $\sqrt{x^{2}+...

Posted by MoMo123 on 21-10-2017 - 00:16 in Bất đẳng thức và cực trị

Cho các số thực dương x, y, z thỏa mãn $\sqrt{x^{2}+y^{2}}+\sqrt{y^{2}+z^{2}}+\sqrt{z^{2}+x^{2}} = 2016$ CMR:

P= $\frac{x^{2}}{y+z}+\frac{y^{2}}{x+z}+\frac{z^{2}}{y+x} \geq 504\sqrt{2}$

Áp dụng bđt $C-S$ , ta có 

$\sum \frac{x^{2}}{y+z} \geq \sum \frac{(x^{2}+y^{2}+z^{2})^{2}}{x^{2}(y+z)+y^{2}(z+x)+z^{2}(x+y)}$

Áp dụng bđt quen thuộc : $2(x+y+z)(x^{2}+y^{2}+z^{2})\geq 3(x^{2}y+y^{2}x+y^{2}z+z^{2}y+z^{2}x+x^{2}z)$

$\rightarrow P\geq \frac{3(x^{2}+y^{2}+z^{2})}{2(x+y+z)}$

Theo đề bài , ta dễ dàng tìm được Min của $x^{2}+y^{2}+z^{2}$, Max của x+y+z Từ đó suy ra $P \geq 504\sqrt{2}$




#694246 $\boxed{\text{TOPIC}}$ Ôn thi học si...

Posted by MoMo123 on 06-10-2017 - 00:24 in Tài liệu - Đề thi

Giải phương trình và hệ phương trình
1) $\sqrt{7x+1}-\sqrt{3x+19}=2x-9$
2) $(\sqrt{x+5}-\sqrt{x+2})(1+\sqrt{x^{2}+7x+10})=3$
3) $\left\{\begin{matrix}\sqrt{x+3y}-\sqrt{x+y-1}=1 \\ \sqrt{x+3y}+x-y=2 \end{matrix}\right.$
4) $\left\{\begin{matrix}(x^{2}+y^{2})(x^{2}-y^{2})=144 \\ \sqrt{x^{2}+y^{2}}-\sqrt{x^{2}-y^{2}}=y \end{matrix}\right.$
5) $\left\{\begin{matrix}2x^{2}+x+y^{2}=7 \\ xy-x+y=3 \end{matrix}\right.$
6) $\left\{\begin{matrix}x-\sqrt{y-1}=\frac{5}{2} \\ y+2(x-3)\sqrt{x+1}=\frac{-3}{4} \end{matrix}\right.$
7) $\left\{\begin{matrix}(x^{2}+1)(y^{2}+1)+8xy=0 \\ \frac{x}{1+x^{2}}+\frac{y}{1+y^{2}}=\frac{1}{4} \end{matrix}\right.$
8) $\left\{\begin{matrix}\sqrt{1-y^{2}}=1-x \\ \sqrt{1-x^{2}}=\sqrt{3}-y \end{matrix}\right.$
9) $\frac{9}{x^{2}}+\frac{2x}{\sqrt{2x^{2}+9}}=1$
10) $\sqrt{x^{3}-3x^{2}}+2\sqrt{(x+1)^{3}}-3x=0$
11) $\sqrt{x^{^{2}}+10x+21} +3\sqrt{x+3}=2\sqrt{x+7}-6$
12) $x^{2}+3\sqrt{x^{2}-1}=\sqrt{x^{4}-x^{2}+1}$
P/S: Thân gửi những bạn có nguyện vọng thi trường chuyên PBC thì làm được hết 12 bài này thì coi như đạt yêu cầu phần phương trình và hệ phương trình nhé. Ấy mà mình chưa làm được câu 11, ai chỉ mình với :(

 

Không biết lên diễn đàn bạn mới trình bày tắt hay ngoài vở bạn đã trình bày đầy đủ rồi nhưng mình thấy tốt nhất là nên làm cụ thể chứ như thế chưa chắc đã giải đến được đích. Vở mình đang cho bạn mượn nên tối về sẽ so sánh với cách giải của bạn. Thứ nhất bài 5 đề đúng bạn nhân PT (2) với 3 rồi trừ vế theo vế để được PT dùng Delta để phân tích thành nhân tử, thứ hai bài 3 bạn chưa giải được cách tổng quát. Theo thầy mình nói thì chỉ may mắn mới giải được bài dạng này nếu không biết cách giải tổng quát.

Câu 5 đúng là mình nhầm thật , còn câu 3, đương nhiên là có cách tổng quát , nhưng ở đây, mình trình bày cách giải nhanh gọn , dễ hiểu nhất nha bạn , giải loại PT này dạng tổng quát chắc ai cũng biết rồi



#693911 Cho a,b,c>0 và a+b+c=1 CMR : $\sum \frac{a}...

Posted by MoMo123 on 29-09-2017 - 18:09 in Bất đẳng thức và cực trị

Mình có cách này không biết có được kinh
$\sum \frac {a}{1+b-a}=\sum \frac {a^{2}}{a (1+b-a)}\geq \frac {(a+b+c)^{2}}{a+b+c+ab+bc+ca-(a^{2}+b^{2}+c^{2})}\geq \frac {(a+b+c)^{2}}{a+b+c}=1$



#693598 $\boxed{\text{TOPIC}}$ Ôn thi học si...

Posted by MoMo123 on 23-09-2017 - 21:30 in Tài liệu - Đề thi

Không cần quá nghiêm khắc thế đâu

Đâu có nghiêm lắm đâu Tea Coffee

Theo như anh Minhnksc đã nói thì bài trên là bài bđt trong đềthi USATST $2001$ đã được đăng tại đây

Tiếp theo là những bài mới cho các bạn nhé

$\boxed{\text{Bài 16}} $(Mình chếtừ bài anh số$6$ Minhnksc đưa ra)

$a_{1},a_{2},...,a_{n}>0$ TM $\sum \frac{1}{a_{1}+1}=n-1$

Tìm Max $\prod a_{1}$ 

$\boxed{\text{Bài 17}}$ Cho $\left\{\begin{matrix}x,y,z> 0 & & \\ xy+yz+zx=1 & & \end{matrix}\right.$

Tim Min $7x^{2}+45y^{2}+64z^{2}$

$\boxed{\text{Bài 18}}$ Cho $\left\{\begin{matrix} x,y,z >0 & & \\ x+y+z=1 & & \end{matrix}\right.$

CMR $\sum \frac{a}{1+bc}\geq \frac{9}{10}$

P/s : Dạo này mình bận quá , không có thời gian lên diễn đàn nhiều, nên làm cho TOPIC bị trì trệ như vậy , mong mọi người vẫn sẽ tiếp tục ủng hộ TOPIC , 

(kiểm tra liên miên)